Hi TootyFrooty!
You're right that this question is difficult. This question could certainly take you a bit longer than average, but I'll walk you through a strategy that would make answering this question as time-efficient as possible.
So, we know that this is a reference question, because it's asking us to choose a phrase that "most accurately describes" a concept from the passage. Therefore, the first thing we want to do is
find where that concept is referenced. In this case, the concept is "the ideal characteristics of an underground area suitable for the deep-well injection of hazardous wastes."
Now that we know that, we can go back and start skimming the passage. Just a few sentences into the very first paragraph, you'll see this section:
In this method, wells are drilled into porous and permeable rock strata that are already saturated with salt water. Liquid wastes are then injected into the rock strata. Most of these wells are drilled to a depth of at least 300 meters—the minimum depth that generally puts the injected waste at a safe distance below any aquifer, in this case a rock stratum containing drinkable water. Such wells are rarely deeper than 1,800 meters, because below this depth it is more cost-effective to consider an alternative method of disposal.
From this section, we know that:
1. Wells are drilled into porous, permeable rock strata that are saturated with salt water
2. Most wells are drilled to a depth of at least 300 meters, because the well needs to be at a safe distance below drinkable water
3. Most wells are not drilled deeper than 1,800 meters
Just using the second two criteria (at least 300 meters, no more than 1,800) we can eliminate answer choices (C), (D), and (E). Answer choice (A) has us drilling into a layer of
impermeable rock; answer choice (B) has us drilling into a layer of
permeable rock. Ignore everything else! We already have our answer—the first paragraph tells us that "wells are drilled into porous, permeable rock." Answer choice (A) has us drilling into impermeable rock; therefore, it can't be right. Answer choice (B) is the only one left, and is the correct answer.
Does that help explain the strategy to attack this question?